You are on page 1of 50

I.

`Introduction
a. Three questions to ask when looking at a Rule of Evidence:
i. What is the rule saying?
ii. What is the purpose of the rule?
iii. Are there alternatives to the current rule?
b. Purpose of the Rules:
i. Accuracy of fact finding.
ii. Mistrust of the jury we dont trust theyll do it right.
iii. Serve substantive policies relating to the matter being litigated
(burdens of proof).
iv. Serve other societal interests (privileges).
v. Control the trial process.
c. How to handle the flow?
i. Judge to be a gatekeeper.
ii. Malpractice against atty/prof. sanctions.
iii. Pre-trial hearings.
d. Rules to Keep in Mind
i. FRE 101: Scope; Definitions
1. (a) refers to the scope; the rules apply to proceedings in U.S.
courts; there are special proceedings that are laid out in
1101.
2. (b)(1-6) definitions of civil case, criminal case, public
office, record, a rule prescribed by the Supreme Court,
and a reference to any kind of written material or any other
medium include electronically stored information.
ii. FRE 102: Purpose
1. Very similar to FRCP 1.
2. Looks toward fairness, eliminating unjustifiable expense,
and the development of law.
iii. FRE 103: Rule of Enforcement
1. An attorney needs to make a motion/objection and the judge
ultimately decides.
2. Puts the burden on the litigants.
3. General Rule: if you dont create a record at trial, thats it.
4. On evidentiary matters, an appellate court can hear an
appeal on an abuse of discretion standard BUT a lot of
deference will be given to the trial judge.
a. Even if the appellate judge disagrees.
iv. FRE 611: Mode and Order of Examining Witnesses and Presenting
Evidence.
1. (a) The court is to exercise reasonable control.
a. (1) power and obligation of the judge.
b. (2) avoid wasting time
i. Tied w/ 403
ii. Obviously concerned w/ time
c. (3) while you can harass/embarrass a witness, still
can look to discredit a witness.
2. (b) Cross-examinations shouldnt go beyond the subject
matter.
a. Also ties in w/ Rule 104.
3. (c) Leading questions are ordinarily not allowed except on
cross-examination and w/ a hostile witness.
a. A hostile witness is someone who: is hostile,
unwilling, or biased; the child witness or the adult
with communication problems; the witness whose
recollection is exhausted; and undisputed preliminary
matters.
II. Logical & Legal Relevance
a. FRE 402: General Admissibility of Relevant Evidence
i. Evidence is admissible unless if not allowed by the:
1. U.S. Constitution;
2. A federal statute;
3. These rules; or
4. Other rules prescribed by the Supreme Court.
ii. Irrelevant evidence is not admissible.
iii. Things of note about the rule: evidence is not defined; states what
is/is not relevant.
iv. Reason for the rules design: trials would go on forever; would
confuse jury; waste courts time.
v. The real question in all of this is what evidence is relevant?
b. FRE 401: Test for Relevant Evidence
i. Overall: Just because something is probative does not necessarily
mean that it is material.
ii. (a) probative/logical evidence: has any tendency to make a fact
more or less probable than it would be w/o the evidence;
1. There needs to be a connection to what youre trying to
prove.
2. To be probative, evidence need not prove anything
conclusively. It merely must have some tendency to make a
fact more or less probably. Evidence will be probative if it
contributes just one brick to the wall of proof built by a
party.
3. Must have a tendency to make the existence of the fact
more probable or less probable then it would be without the
evidence.
4. Problem 1.1 p. 25 Show me the body
a. This rule indicates that the wife knows where the
body is.
b. Maybe she also knows they dont have it (because,
again, she knows where the body is).
c. Maybe she knows more about the murder than she
led on.
5. Problem 1.2 p. 26 Brotherhood
a. If the defendant objects to the prosecutors question
of Mills as irrelevant, the court should find the
evidence relevant. As being a member of the prison
type of organization, it is more likely than not that
Mills would lie for another member or the
organization (in this case the defendant) and that
Mills statements about what Ehle said were false. It
can also be inferred that Ehle is not a member of the
organization.
iii. (b) materiality of material/factual evidence: the fact is of
consequence in determining the action
1. Whether evidence is material turns on what issues are at
stake in the proceeding, which often turns on the
substantive law of the jurisdiction.
a. LOOK AT THE SUBSTANTIVE LAW TO DETERMINE!!!!!!
2. Does it bear of a fact of consequence to the determination
of the action? Often depends on the substantive law.
3. Problem 1.4 Knowledge (of prior conviction w/ loaded gun)
a. Even though this is not about probativeness, even
though she did not know the length, knew she was
convicted.
i. If this is true, she didnt know she was
breaking the law.
b. Materiality: we dont care whether or not she knew;
she was ultimately breaking the law.
i. The statute tells us here its not a fact of
consequence.
ii. The substantive law is ultimately needed to
show materiality.
4. U.S. v. James
a. Facts: James appealing conviction to aiding and
abetting by giving her daughter a gun that ultimately
killed decedent. At trial, the evidence to corroborate
the defendants story about the decedents past was
not let in by the judge when the jury asked for it and
James was convicted.
b. Issue: Whether or not the trial judge erred by not
allowing the evidence of the decedents record into
evidence?
c. Holding: Yes, the appellate court holds that the trial
judges view of the evidence was too narrow and led
directly to the defendants credibility so the decision
was reversed.
5. Knapp v. State
a. The converse of James.
b. The defendant complained that the trial court
permitted the state to prove that the stories the
defendant claimed to have heard were not true.
c. Just as Knapps unnamed sources were more likely to
say the old man was clubbed to death if he really
was, so Ogden was more likely to say he robbed an
old man at knifepoint if he really did.
iv. FRE 401 is an extremely low bar BUT
1. A proposed generalization does not equal guilt.
2. Instead, the apparent probability is greater than before the
evidence.
3. Low standard of admissibility.
c. FRE Structure
i. Section IV deals with evidence that may be too prejudicial
even though it is probative.
ii. Section V recognizes a number of privileges which exclude
otherwise relevant evidence.
iii. Section VI imposes limits on witnesses and the manner in
dealing with them.
iv. Section VII specifies requirements with respect to opinions and
expert testimony.
v. Section VIII excludes hearsay that does not fall within an
exception.
vi. Section IX spells out the handling and authentication of evidence.
1. *** bold are the main focus of our class***
d. FRE 104: Preliminary Questions
i. Really sets out the roles between the judge and the jury.
ii. The judge is ultimately the gatekeeper.
iii. FRE 104 (a)
1. Judge determines if the evidence meets a condition
a. Ex. A witness saying that the evidence is protected
by the atty-client privilege.
2. A judge needs context.
3. Both a legal and factual decision.
4. In making their decision, the judge is not bound by FRE
except privilege.
5. Not done in front of the jury.
iv. FRE 104 (c)
1. Instances where the jury cannot hear
v. FRE 104 (b): Conditional Relevance
1. Cox v. State
a. Facts: Cox shoots Leonard while Leonard was asleep;
Leonard dies 3 days later; Coxs friend Hammer in
prison for pending charges regarding molesting
Leonards daughter; Hammers mom was there at the
hearing for Hammer but Cox was not; Cox was a
close family friend and stayed w/ the mom; Cox was
found guilty and he claimed the evidence was not
relevant because he would have needed to know
about it.
b. Issue: Whether or not the trial court erred by
admitting certain testimony, the relevance of which
depended upon Coxs knowledge of the content of
the testimony?
c. Holding: The court holds that the evidence admitted
was sufficient to support the inference that Cox had
learned what transpired at Hammers hearing and
the court therefore did not abuse its discretion by
admitting the evidence.
2. Under its terms, the court may admit the evidence only after
it makes a preliminary determination that there is sufficient
evidence to support a finding that the conditional fact
exists.
a. In Cox, the evidence was that the mother was at the
hearing and Cox was a close family friend always at
the home.
b. The judge decided that a reasonable jury could
conclude this evidence could be established at trial.
3. Overall, the judge is the gatekeeper who hears it first THEN
decides if it would go to the jury.
a. Could a reasonable jury conclude by a
preponderance of the evidence?
b. Conditional relevance might be relevant but only if
some other condition is met.
c. Sufficient evidence is required to support a jury
finding of the conditional fact.
4. Lawyers do not make many conditional relevance objections
because while it is higher than the bare relevance standard,
it is not much higher.
5. If Rule 104(b) applies, the test is: could a reasonable jury
infer the fact by a preponderance of the evidence.
a. But if 401 applies, test is any tendency standard.
b. Which is easier to meet?
i. 401 is easier to meet but both are pretty low
standards.
e. FRE 403: Excluding Relevant Evidence for Prejudice, Confusion, Waste of
Time, or Other Reasons
i. The Court may exclude . . .
1. Up to the judge w/ respect to review.
ii. Relevant evidence . . .
1. The evidence being excluded was actually relevant.
iii. If its probative value is substantially outweighed by . . .
1. Extremely liberal.
2. This is a balancing test by the court.
3. The probative value will outweigh the prejudicial value
4. There is a presumption under 403 w/ some issues that it will
be admissible.
5. Substantially outweighs favors admissibility.
iv. A danger of . . . unfair prejudice . . .
1. Relevant evidence is only unfair prejudice when it
substantially outweighs the probative value, which permits
exclusion of relevant matter under Rule 403.
2. Unfair prejudice within its context means an undue
tendency to suggest decision on an improper bias,
commonly, though not necessarily, an emotional one.
3. Prejudice vs. unfair prejudice
a. Unfair when it really goes over.
4. Always want to raise FRE 403 and unfair prejudice.
a. Has an umbrella function to it.
v. A danger of . . . confusing the issues, misleading the jury . . .
1. Distracting the jury from the task at hand.
vi. A danger of . . . undue delay, wasting time, or needlessly
presenting cumulative evidence.
1. All considered as wasting the courts time.
vii. State v. Bocharski
1. Facts: court admits exhibits 42-47 into evidence.
2. Issue: whether or not all of the photos should have been let
in?
3. Holding: the court allows all photos except 46 and 47 due to
unfair prejudice.
viii. Problem 1.8 p. 48 Photo of Guns
1. Would get in under 401.
2. Would unfairly prejudice the jury under 403; all the guns
dont belong to him and it would make the jury believe that
hes a gun nut and would want a machine gun.
ix. 403 Questions
1. Could the jury evaluate rationally?
2. Alternative means of proof?
3. What is at issue?
4. How close is the link?
5. Can evidence be made less unfairly prejudicial?
6. What are the parties really fighting about? (do we really
need it?)
7. What is the specific harm?
8. What inferences?
9. How is the evidence being presented?
x. U.S. v. Myers
1. Facts: defendant fled from a FL mall when his g/f was
bringing him clothes and he saw the cops; in CA the cop
nudged his bike to get him.
a. The logical linkage of evidence on a flight instruction:
i. defendants behavior to flight
ii. from flight to consciousness of guilt
iii. from consciousness of guilt to consciousness
of guilt for crime charged
iv. to actual guilt of crime charged
2. Issue: Whether or not the defendant was fleeing the police
due to the robbery when it was unclear that the defendant
committed the robbery?
3. Holding: In order to get the instruction in all four were
needed and since number 2 and number 3 are not there,
then the ruling was reversed.
xi. Problem 1.11, p. 65 Staying Put
1. This is a problem of the inverse of running/flight (i.e. not
running when one has the means to).
2. Does it work on the flip side?
a. Courts usually dont worry so much about prejudice
to the prosecution so it would probably be let in.
xii. Old Chief v. U.S.

1. Facts: In 1993 Johnny Lynn Old Chief was arrested after a


fight involving at least one gunshot. Old Chief was charged
with, inter alia, violating 18 U.S.C. 922 (felon in possession
of a firearm) and aggravated assault. Old Chief had been
previously convicted of assault causing serious bodily injury.
In order to prove his status as a felon in possession the
prosecution wanted to introduce information about the prior
felony conviction, specifically what Old Chief was convicted
of and what he was sentenced to. In order to keep this prior
conviction from the jury, Old Chief offered to stipulate that
he was previously convicted of a crime punishable by
imprisonment exceeding one year, but not to further identify
the nature of the prior conviction. The government refused
to join in a stipulation.
2. Issue: Whether or not the trial court abused its discretion if
it spurns such an offer and admits the full record of a prior
judgment, when the name or nature of the prior offense
raises the risk of a verdict tainted by improper
consideration, and when the purpose of the evidence is
solely to prove the element of prior conviction?
3. Holding: The court holds that it does because the
stipulation would serve the purpose the government needed
to prove that there was a violation of 18 U.S.C. 922.
4. ***This case is a special circumstance; in dicta, court says
its important for govt. to prove their case.***
a. REMEMBER, one piece of evidence can prove more
than one thing.
f. FRE 105: Limiting Instruction
i. Must restrict the evidence to its proper scope and instruct the jury
accordingly.
ii. w/ Old Chief
1. What would you instruct the jury: only able to use evidence
for firearms charge; not assault.
2. When would you give it: judges have a great deal of
discretion but an instruction can be given when the
evidence is introduced but will also definitely be done at the
end of trial.
3. Concern: jury wont consider the instruction and use
evidence for assault.
iii. w/o FRE 105, there would be a lot of excluding evidence under FRE
403.
III. Special Relevance Rules
a. Notes about Rules 407-411
i. Two Ways to Think About FRE 407-411 Rules
1. They reflect the rule-makers judgment about how FRE 403
analysis should come out.
2. They are unnecessary and redundant, since FRE 403 should
lead to the same result.
ii. The structure of the rules:
1. FRE 407-409, 411 are rules of limited exclusion: they
exclude evidence for specific reasons, but allow for all other
purposes.
2. In contrast, FRE 410 is a rule of expansive exclusion. It
excludes plea discussions for all purposes but two.
b. FRE 407: Subsequent Remedial Measures
i. Evidence cannot be admitted to prove negligence, culpable
conduct, a defect in product/design, or a need for a warning on an
instruction.
ii. However, the evidence could be brought in for another purpose
such as impeachment, proving ownership control (if disputed), or
the feasibility of precautionary measures.
iii. Problem 2.1 p. 99 Wolf Attack
1. Beagle owner: would not be able to get the evidence in if
looking to get it in under a negligence theory. The owner
could use it if it made things better but it would be used
against him so unlikely to be used.
a. From a 401/403 argument, doesnt mean he was bad
yesterday (and this would not necessarily keep it out
though).
2. Childs parents: They would make a feasibility argument
and would likely get the evidence in because it is not a
subsequent, remedial measure: the owner already took the
measure by chaining the wolf. Plus, the owner was already
on notice of the wolfs conduct.
iv. Tuer v. McDonald
1. Facts: Mr. Tuer had an angina problem and was taken off
Heparin before his scheduled surgery; was rescheduled for
later on in the day due to his doctor attending another
emergency; Heparin not restarted and Mr. Tuer eventually
died due to complications. After this incident the hospital
changes their Heparin policy.
2. Issue: Whether or not the trial court erred in excluding
evidence that, after Mr. Tuers death, the defendants
changed the protocol regarding the administration of the
drug to patients awaiting coronary artery bypass surgery?
3. Holding: The court holds that the change in policy would
not be admissible and the doctors feeling that the policy
was correct was not impeachable.
a. Change in procedure/feasibility of administering
drug.
i. Defense experts say what this doctor did was
proper protocol.
ii. Plaintiff would argue that it was feasible to
give medication.
iii. Expert/defense says the risk outweighed the
benefit.
b. Impeachment
i. Deciding at some point the Heparin was now
safe.
ii. However, before they said it was unsafe.
iii. Impeachment usually allows evidence in,
however, here it wasnt because it was based
on information he had at the time.
v. Feasibility
1. Narrow View
a. Feasibility is not controverted and thus subsequent
remedial evidence is not admissible under the Rule.
2. Broad View
a. Feasibility means more than that which is merely
possible, but including that which is capable of being
utilized successfully.
c. FRE 408: Compromise Offers and Negotiations
i. This rule applies to both parties
ii. It is a broad prohibition that doesnt offer protection before a claim;
cant be used to impeach.
iii. From a policy standard, we want people to settle and not go to trial;
in essence, we want a system that encourages settlement.
iv. w/o this policy, there would be a disincentive to enter negotiations
or talk about settlement.
v. Doctors and saying Im sorry: this usually becomes before a claim
is filed and thats why they dont say it.
d. FRE 409: Offers to Pay Medical and Similar Expenses
i. Under the same breath of FRE 408.
ii. We want a system that encourages settlement.
iii. The offer could be given out of kindness and not due to guilt/fault
iv. Contrary to Rule 408, dealing w/ offers of compromise, FRE 409
does not extend to conduct or statements not a part of the act of
furnishing or offering or promising to pay.
e. FRE 410: Pleas, Plea Discussions, and Related Statements
i. Broad in protections but ONLY protects the defendant.
ii. Policy reasons: relevancy reasons for innocent people to plead;
financial considerations; want prosecution and defense to speak
openly (efficiency).
1. Keeping in mind the vast majority of criminal cases settle.
iii. Problem 2.8 p. 137 Prosecutors Offer
1. Does 410 exclude statement?
a. No wouldnt bar prosecutors case.
2. Should be able to get it in under FRE 403 but it could also
mislead jury/confuse issues.
iv. U.S. v. Biaggi
1. A plea rejection might simply mean that the defendant
prefers to take his chances on an acquittal by the jury,
rather than accept the certainty of punishment after a guilty
plea.
2. The probative force of a rejected immunity offer is clearly
strong enough to render it relevant.
3. The court concluded that in this case the exclusion of
Mariottas state of mind evidence denied him a fair trial.
4. Where evidence of a defendants innocent state of mind,
critical to a fair adjudication of criminal charges, is excluded,
we have not hesitated to order a new trial.
v. Statements from plea negotiations cannot be used to impeach.
f. FRE 411: Liability Insurance
i. Evidence of an individuals actual/lack of insurance is not
admissible but it can be entered into evidence for another purpose
such as proving witness bias or prejudice or proving agency,
ownership, or control.
1. Generally speaking, juries are more likely to rule positively
for a plaintiff when the defendant has insurance.
ii. Problem 2.5 p. 130 Claims Adjuster
1. The statements made by the insurance agent allowed in.
2. The plaintiff would want these statements in to show a
financial imbalance.
3. The rule does not want the jury just to stick to insurance.
4. Why would it be useful for the jury to know the information
from the insurance?
a. Because the adjuster is representing the insurance
company.
b. In the problem, they used a recording.
iii. The rule only prohibits proof of insurance when proving negligence.
1. Can use it for anything else!
2. The rule would not prohibit questioning.
IV. Character Evidence
a. In general
i. Can have a predictive quality; tells us what someone will do when
nobodys watching.
1. Not to the extent of habit
ii. It also shows us what kind of person someone really is.
iii. Ask friends and family
iv. Test: go to 404 first, then go to 405.
v. 2 ways character questions can arise
1. character in issue: may be element of crime, claim,
defense
2. circumstantial: suggesting an inference that the person
acted consistently w/ character trait.
b. FRE 404(a)(1)
i. In essence says you cannot use character evidence to prove
propensity.
ii. Commentary: Character evidence is of slight probative value and
may be very prejudicial. It tends to distract the trier of fact from
the main question of fact from the main question of what actually
happened on the particular occasion. It subtly permits the trier of
fact to reward the good man and to punish the bad man because of
their perspective characters despite what the evidence in the case
showed actually happened.
iii. Have to find a way to go around the propensity box
iv. People v. Zackowitz (1930)
1. Facts: Zs wife goes to street corner and gets insulted; goes
to tell Z; Z comes back and says if they dont get out of
there hell bump them all off; went back to house all heated
and intoxicated; rage infuriated him and he took one of his
guns (which was allegedly with him all night) went back and
shot one of the guys in the stomach killing him; threw gun in
the river; caught 2 months later.
2. Issue: whether or not Zs actions were deliberate or
premeditated (ultimately looking at the state of mind of Z).
3. The evidence being fought over was Zs array of guns.
a. Prosecution wanted to show Zs murderous
disposition.
b. Character in slides (to prove vicious and dangerous
character)

c. 2 things are going on here


i. Preventive justice/conviction
ii. Gives excessive weight
d. The logic works w/ character evidence but it
becomes too relevant for the jury.
4. Ultimately, the character evidence was not allowed.
5. Would have need to find a way around the propensity box:
v. One easy way to think of character evidence, as shown above: the
propensity box
1. You ultimately cannot go through it.
2. Have to go around it (w/ respect to Z, it could be that he
went back and chose the gun).
3. If it got in, can always have a limiting instruction w/ FRE
105.
c. FRE 404 (b)
i. This is NOT an exception to the character rule under 404(a)(1).
1. FRE 404(b)(1) simply lists prohibited uses (its superfluous
and as 401 says this)
2. FRE 404 (b)(2) can be used in either a criminal or civil
case and is not an exhaustive list: ultimately may let
evidence in for another purpose
a. 2 additional ones are doctrine of chances and res
gestae.
ii. 3 step analysis
1. Sufficient evidence that prior incident occurred. How
much is needed? (Huddleston = Preponderance)
2. What purpose is the evidence being offered for? Is
the purpose proper and is this evidence relevant?
3. Probative worth vs. unfair prejudice.
iii. Example: video w/ military person and gun; not using it for
character but knowledge that he knew how to use weapons.
1. When going to a 403 balancing, going to look for similarity
of weapons and context
iv. Problem 3.1, p.165 - Hacker
1. To get around propensity, similarity of plan and identity.
2. Draw the link between the 1st and 2nd crime.
3. Unfairly prejudicial would be a good counter against id/plan.
v. Problem 3.2, p. 166 Drug Seller
1. Questions to ask: type of drug? Where is occurred? How
transactions happened? How long ago did it happen?
vi. Things to keep in mind w/ 404(b)
1. Not an exhaustive list
2. Includes other crimes, wrongs, or acts doesnt have to be a
crime
a. Frequently brought up in criminal cases by
prosecution.
3. There are notice requirements in the criminal context.
vii. Video clip (woman killed; body not found)
1. A mix of going through and around the propensity box.
2. On a 403 balancing:
a. For prosecution: defendant can get rid of body w/o a
trace, therefore not unfairly prejudicial.
b. For defense: inflame the jury; not enough info; b/c
he has knowledge doesnt mean hes a bad person.
c. For prosecution: he possesses the knowledge
viii. Knowledge, in general is a fine line.
ix. Problem 3.4, p.167 U.S. v. Peltier I
1. Crime charged w/ is murder.
2. Trying to introduce skipping bail.
3. Helps prove motive he may possibly see them as officials.
x. Problem 3.6, p169 U.S. v. Peltier II
1. Peltiers weapon (AR-15)
a. The other crime was possession (obtained after the
crime).
b. 1. They were in his trailer; possession.
c. 2. Character risk: violent disposition; Non-character
risk: theory; opportunity; knowledge; identity (is the
strongest)
d. 3. Pro: no other way to prove identity; Con: no way
to know he got the weapon at the time; inflaming the
jury.
2. Victims weapon w/ Ps thumbprint
a. The other act is possession of a victims weapon.
b. 1. Sufficient evidence was there.
c. 2. Char risk: jury assuming P killed agent; non-char:
identity
d. 3. Char risk would be risk but allowed in.
3. Grenades, slips of paper w/ code, etc.
a. 1. Sufficiency there.
b. 2. Character evidence: bad person; he may do more;
non-character risk: knowledge of how to use a
weapon
c. 3. Character risk would probably outweigh non-
character based on other evidence; likely prejudicial.
xi. Modus operandi
1. So specific/unique to themselves it cannot possibly be
anyone else.
xii. Other routes around the propensity box
1. Narrative integrity (res gestae): more of a story needed
2. Absence of mistake
xiii. U.S. v. Trenkler (1995)
1. Facts: 2 bombings: one in Quincy, the other in Roslindale;
looking to show modus operandi; lower court found the
defendant guilty.
a. Similarities: bomb switch the same; done for a
friend; both attached to a vehicle; remote control
detonated.
b. Differences: nothing special about detonator; one in
a box, the other duct tape; material different;
batteries different
2. Issue: Whether or not the lower court erred when finding
that the two crimes were similar?
3. Test:
a. Special relevance independent of its tendency simply
to show criminal propensity.
b. If special relevance is on a material issue, then 403
analysis.
4. Conclusion: The court upheld the conviction although, as
the dissent shows, could have easily gone the other way.
xiv. Problem 3.10, p. 189 Russian Roulette
1. Issue at trial: is weapon the same from a prior occasion?
2. Non-char risk: identity; char. risk: reckless and using gun
for Russ. Roulette; how would witness know it was the same
weapon?
3. More needed to show why witness knows so much about
gun (res gestae).
xv. Problem 3.11, p. 193 Cleaning His Gun
1. Non-char: lack of accident (he should have known how to
clean gun).
xvi. Additional hypos
1. Defendant charged w/ ingestion of pot.
2. Defendant claims he went to a party and innocently at a
plate of brownies laced w/ pot.
3. Govt. wants to introduce evidence defendant had been
acquitted last year of illegal drug use and he used same
defense
a. Doctrine of chances theory.
xvii. A few more points on 404(b)
1. Does not have to be a crime and does not have to be similar
to the charged offense depending on relevance of the
evidence.
2. Can include incident post facto.
3. Both sides can suffer unfair prejudice but unfair prejudice to
the govt. is less likely.
4. Most often used by the prosecution in rebuttal or to blunt a
suspected claim by the defense.
5. Rule has notice requirements.
6. California has an exception allowing character evidence in
(think of O.J. beating his wife).
d. Rules 413-414 (allowing propensity to be shown)
i. Evidence of Similar Crimes in [413. Sexual Assault] and [414. Child-
Molestation] Cases
1. In a criminal case in which a defendant is accused of a
sexual assault, child molestation, the court may admit
evidence that the defendant committed any other sexual
assault. The evidence may be considered on any
matter to which it is relevant.
2. 415 is similar but that is for civil cases.
ii. Policy reasons for this:
1. Make case easier for prosecution
2. Recidivism (makes propensity argument stronger)
iii. U.S. v. Guardia (1998)
1. Facts: defendant charged w/ 2 counts of sexual assault due
to incidents during several alleged incidents during a gyno.
visits. 4 other women also ready to testify; lower court
excluded the testitmony.
2. Issue: on the motion in limine, whether or not the other 4
women should be excluded from testifying?
3. 3 requirements under FRE 413
a. Defendant accused of an offense of sexual assault.
b. Evidence proffered is evidence of defendants
commission of another offense of sexual assault.
c. Relevancy.
4. Finally, must to a 403 analysis (relevant factors)
a. Similarity of crimes; time; frequency; need.
5. Holding: Found the lower courts interpretation correct and
the other 4 womens testimony should be excluded.
e. FRE 412 (rape-shield rule)
i. Pre-rape shield law instructions really throwing propensity in.
1. Issue is that evidence could be used as relevant to consent.
2. Credibility as well based on prior activity.
3. In broad terms, the truthfulness of consent doesnt really fit
401
4. Concern of this is of a likelihood of a victim would not come
forward.
ii. 412(a) Prohibited uses
1. Very broad; acts, mode of dress
iii. 412(b)(1)(A)-(C) Exceptions in a criminal case
1. (A) can be offered to show someone else.
2. (B) consent in the past w/ defendant admissible.
3. (C) violate defendants constitutional rights (ex. When victim
in past untruthful about rape/false reporting).
a. Another examples: a prostitute claimed rape when
solicitor wouldnt pay.
iv. 412(b)(2) Civil Case
1. A balancing test
f. FRE 404(a)(2)(A)
i. (2) Exceptions are for a Defendant or Victim in a Criminal Case
ii. (A) A defendant may offer evidence of defendants pertinent trait,
but if its admitted, prosecution may rebut.
iii. Ultimately, defendant can open the door to his own character trait.
iv. Michelson v. U.S. (1948) pre-rules case
1. Facts: defendant convicted of bribing a federal revenue
agent; 5 character witnesses were called; defendant said he
wouldnt have done it on his own. Two of the witnesses
heard about his prior crimes, 2 hadnt.
2. Issue: Question on appeal is whether there was reversible
error?
3. Holding: The court affirms the judgment.
v. Problem: Don is charged w/ assaulting Vince by hitting him in the
face with a shot glass, causing cuts to Vs face. Don claims self-
defense and there is some dispute over who started the
confrontation, even though the evidence suggests that Don threw
the first punch.
1. Question 1: Prosecution wants to call Coach Jones to testify
that Don is a mean, aggressive, and physical man, quick
tempered, and prone to violence
a. Not allowed because a character trait cannot be
brought up (404(a)(1)).
2. Question 2: Defense wants to call Reverend Graham to
testify that Don is a peaceful person toward all people,
gentile and non-violent, and not likely to initiate violence.
a. Assuming its a criminal case, then allowed under
404(a)(2)(A).
3. Question 3: Assume that after defense calls the Reverend,
Prosecution then calls Coach Jones?
a. Yes, it would be allowed under 404(a)(2)(A) because
defendant opened the door first.
vi. Character Trait of the Accused
1. Accused holds the key.
2. Pertinent character trait
3. Once the door is open the prosecution can rebut w/ contrary
character evidence.
g. FRE 404(a)(2)(B)&(C)
i. (2) Exceptions are for a Defendant or Victim in a Criminal Case
ii. (B) subject to Rule 412 limitations, a defendant may offer evidence
of a victims pertinent trait and if admitted, the prosecutor may
1. (i) offer evidence to rebut and
2. (ii) offer evidence of the defendants same trait
iii. (C) Prosecutor can offer victims peacefulness in a homicide case to
rebut evidence that the victim was the first aggressor.
iv. Bar fight hypo continued
1. Question 4: Don calls Ernie to testify that Vince is a
belligerent fight picking, aggressive person with a real
short fuse.
a. Yes, under 404(a)(2)(B)
2. Question 5: Assuming the judge allows Ernie to testify, can
the prosecution introduce testimony from Jen that Vince is a
calm and peaceful person who does not have a violent bone
in his body?
a. Yes, under 404(a)(2)(B)(i)
3. Question 6: Assuming that the judge allows the testimony
by Ernie, can the prosecution now offer testimony from Gary
that Don is a violent and aggressive person?
a. Yes, under 404(a)(B)(ii)
4. Question 7: Assume that instead of calling Ernie, Don calls
eyewitness Wendy to testify that Vince threw the first blow.
Can Vince call Jen to testify that Vince is a peaceful person?
a. No, this goes back to FRE 404(a)(1)
v. Character of the Victim
1. Accused holds the key
2. Pertinent character trait of the victim
3. Victims character in homicide case.
h. FRE 405(a) Methods of Proving Character: By Reputation or Opinion
i. NOT A RULE OF ADMISSIBILITY
ii. (a) By Reputation or Opinion
1. (language of the rule) When evidence of a persons
character or character trait is admissible, it may be proved
by testimony about the persons reputation or by testimony
in the form of an opinion. On cross-examination of the
character witness, the court may allow an inquiry into
relevant specific instances of the persons conduct.
2. All of these are testimony either by reputation or opinion.
a. Reputation is practically hearsay; opinion is based on
witnesss knowledge.
3. Reputation/opinion (how to do it)
4. On cross, can go to specific instances (used for how well
witness knows defendant)
iii. Opinion Testimony
1. Witness Knows the Accused/Victim
2. Witness Has Sufficient Contact to From an Opinion
3. Witness has an Opinion of the Accused/Victims Character
4. Witness State that Opinion: Vince is a belligerent, fight-
picking fellow with a real short fuse.
iv. Reputation Testimony
1. Witness is a Member of the Accused/Victims Community
2. Witness has been a Member for Substantial Time
3. Accused/Victim has a Reputation
4. Witness knows Reputation
5. Witness States Reputation: Vince is known in the workplace
as a belligerent person and people think he is aggressive.
v. 405(a) specific instances on cross
1. Only thing you can ask a witness about is the trait they
know something about.
2. Form: Do you know? Have you heard?
a. Reason; going to knowledge/reputation; only asking
for 1 reason: to test witnesss knowledge.
3. Specific instances must relate to the character trait being
discussed.
4. Good faith basis: an arrest doesnt equal
conviction/wrongdoing; courts will see arrest differently but
you need some sort of proof it occurred.
5. Stuck w/ answer: if not, would really confuse issues.
6. No guilt assuming questions (of the crime currently being
charged with).
i. FRE 405 (b) Methods of Proving Character: By Specific Instances
i. (b) when the persons character or character trait is an essential
element of a charge, claim, or defense, the character or character
trait may also be proved by relevant specific instances of the
persons conduct.
1. A method of proof and when you can use certain types of
evidence.
2. When trait is essential, then it can be used.
3. Applies in civil and criminal cases.
ii. When it arises:
1. (civil) defamation, negligent entrustment, good parenting
2. (criminal) entrapment but very rare otherwise
iii. Most of the confusion stems from an overly broad conception of
what it means for a character trait to be an essential element of a
charge, claim, or defense.
1. Works in a self-defense case because youre looking at the
accused fear and whether or not it was reasonable when
they killed.
iv. Problem 3.15 p. 249 I wouldnt shoot anybody.
1. 405(a)
v. Problem 3.16 p. 250 Character of Victim I
1. Talking about Vs violent nature
2. 404(a)(2)(B) get it in, then go to 405(a)
3. Issue here is that its a specific instance, not a reputation or
opinion.
vi. Problem 3.17 p. 250-251 Character of Victim II
1. 404(a)(2) satisfied
2. 405(b) she was reasonable in her fear
j. FRE 406: Habit; Routine Practice
i. Witness doesnt need to have knowledge of the specific instance;
non-thinking, automatic, predictive conduct
1. Regular drinking NOT a habit; only occasionally admitted.
ii. Doesnt apply much in criminal context; more likely to be used in
civil case.
iii. Problem 3.19 p. 256
1. Not likely to work.
iv. Hilloran v. Virginia Chemicals Inc. (1977)
1. Facts: plaintiff had an accident due to cans of refrigerant;
plaintiff won. At trial, defendant offered a witness to show
that plaintiffs method was done all the time but it was
objected to and sustained.
2. Issue: Whether or not the evidence should have been
allowed in?
3. Holding: the court holds that the judgment should be
reversed as the jury should not be precluded from hearing
such evidence.

V. Impeachment
a. Nonspecific Impeachment
i. ***no rules on bias/capacity in Federal Rules***
ii. Impeachment mechanisms suggest reasons to doubt the witness
but not a direct attack on any particular point of the witnesss
testimony
1. Bias of influence (youre lying)
a. Few Good Men example: witness has an interest in
the outcome.
2. Defect in capacity of the witness (Youre Wrong)
a. My Cousin Vinny example: cook wrong on this
occasion
3. Untruthful character of the witness (Youre a liar)
a. Character witnesses (FRE 608)
b. Cross examination into non-conviction misconduct
related to character for untruthfulness (FRE 608)
c. Cross examination into certain criminal convictions
(FRE 609)
b. Specific Impeachment
i. Attacks a particular point in the testimony of the witness but dont
provide reasons for those shortcomings.
1. Prior inconsistent statement (FRE 613) (Youre lying and
youre a liar).
a. FRE 613 Witnesss Prior Statement
i. (a) Showing or Disclosing the Statement
During Examination.
ii. (b) Extrinsic Evidence of a Prior Inconsistent
Statement.
2. Contradiction (Youre lying and maybe youre a liar)
c. FRE 404(a)(3): Exceptions for a Witness: going to FRE 607, 608, 609
i. For a witness or a defendant who is acting as a character witness.
ii. No limitation to civil/criminal; applies in all cases.
iii. Puts character for truthfulness at issue.
d. FRE 607: Who May Impeach a Witness
i. ANYBODY
e. FRE 608(a): A Witnesss Character for Truthfulness or Untruthfulness (a)
Reputation or Opinion Evidence
i. The window of truthfulness opened as soon as a witness is put on
the stand.
ii. To counter attack, can bring own witness
iii. However, one cannot bolster: ergo, bringing a witness to bolster
truthfulness prior to truthfulness being attacked.
iv. Video: reputation in community
1. Go to 404(a)(3), then 608(a).
2. Says reputation is terrible could be problematic
3. More specific about truthfulness/veracity approved.
4. Would you believe the witness under oath? Question should
be do you know? Have you heard?
v. Bolstering: cant bolster before an attack (608(a))
1. Has to be a direct attack for truthfulness tested (608 or 609)
2. If indirect, cant bolster.
f. FRE 608(b): A Witnesss Character for Truthfulness or Untruthfulness (b)
Specific Instances or Conduct
i. (language of rule) Except for a criminal conviction under Rule 609,
extrinsic evidence is not admissible to prove specific instances of a
witnesss conduct in order to attack or support the witnesss
character for truthfulness. But the court may, on cross-
examination, allow them to be inquired into if they are probative of
the character for truthfulness or untruthfulness of:
1. (1) the witness; or
2. (2) another witness whose character the witness being
cross-examined has testified about.
3. 608(b)(2) (for witness) is like 405 (for defendant)
ii. Hypo: Eyewitness testifies that she saw defendant enter the
convenience store w/a gun. On cross:
1. Defense wants to ask witness about an incident last year
involving drunk driving.
a. Isnt relevant, but doesnt work under 608(b) even if
relevant.
2. Defense wants to ask witness about falsifying her job
application and puffing her resume.
a. Should be allowed under the rule.
iii. Limitations: can cross-examine a witness, but youre stuck w/ the
answer.
1. Judges discretion
2. Cross-examination only
3. Focus only on character trait of veracity
4. Bound by the answer.
iv. Theft wouldnt work but embezzlement would.
v. Remember, questioning must be in good faith.
vi. Problem 4.1 p. 269
1. No; needs to be about truthfulness under 608(a)
a. 404(a)(1) + 404(a)(2) wouldnt apply b/c this is a civil
case.
2. No; bolstering under 608(a) not allowed.
3. No; 6089b) allowed on cross-exam; extrinsic evidence not
allowed except for a criminal conviction.
4. Yes; under 608(a) on direct examination; Rice already
testified.
5. No; good for 404(a)(3) and 608(a) but doesnt work under
608(b) truthfulness.
6. Yes; probative of truthfulness.
g. FRE 609: Impeachment by Evidence of a Criminal Conviction
i. Purpose: permits impeaching a witness, including the defendant,
with evidence of a felony conviction, or with evidence of any
conviction involving dishonesty or a false statement, so long as
conviction is less than 10 years old.
ii. Premised on: felons are liars; if youve been convicted of a crime
w/ element of truthfulness then thats really relevant.
iii. Extrinsic evidence, going back to 608(b), would be allowed in.
iv. In 609(b)(1), presumption is its inadmissible (opposite of 403)
v. Five factors to keep in mind (from Brewer)
1. The nature of the crime
2. The time of conviction and the witness subsequent history
3. Similarity between the past crime and the charged crime
4. Importance of defendants testimony
5. The centrality of the credibility issue
vi. If the defendant offers evidence of a past crime, after losing in a
motion in limine, he cant appeal the judges ruling when allowed to
bring it up
vii. Breaking it down
1. 609(a)(1)
a. a. 609(a)(1)(A): Witness with felony conviction,
subject to 403 (presumption in favor of admissibility)
(Not allow the question unless the judge determines
that the prejudicial effect of the question is
outweighed by the probative value)
b. b. 609(a)(1)(B): Defendant-Witness with felony
conviction, subject to special balancing test
(probative value > prejudice) (prosecution has the
burden of proof for defendant-witnesses)
2. 609(a)(2)
a. 609(a)(2): All witnesses/defendants: Any crime
involving dishonesty/false statements shall be
admitted.
b. Felony or misdemeanor
c. No discretion, no balancing
viii. Video w/ convictions
1. Arson (convicted but not yet sentenced)
a. (1) conviction of more than a year doesnt matter if
he serves less than a year.
b. (A) 403 balancing
c. **good to use here because felons are liars**
2. Conviction 12 years ago; released 8 years ago
a. 10 years have passed since the time of conviction
but his release is less than 10 years so he can be
impeached.
3. Defendants on stand; being cross-examined on arson w/in
10 years and wants to ask
a. 609(a)(1)(B) you presume prejudice
b. Go back to Brewsters 5 factors
4. Perjury
a. 609(a)(2) quintessential example of an untruthful
conviction crimin falsi
b. Doesnt distinguish between a defendant and a
witness; but a very narrow rule.
c. No 403 balancing required: has to come in.
5. Larceny
a. Not under 609(a)(2)
6. Juvenile conviction
a. 609(d)(1-4)
b. Generally, they stay out (very seldom to see it come
in)
h. Impeachment By Prior Consistent Statements (Rule 613) ***Important for
Hearsay***
i. If he said a on day 1, b on day 2, how can he be trusted (613(b))
ii. No notice requirement (613(A))
iii. Can always call cop or obtain other extrinsic evidence
i. Impeaching With Prior Inconsistent Statement (not in rules)
i. Example w/ car accident case, plaintiffs witness testifies as follows:
1. Defendant caused the accident by backing into plaintiff;
eyewitness saw accident from the curb and didnt know the
plaintiff prior to the accident; eyewitness was just leaving
the drug store at the time of the accident.
a. The defendant now wants to introduce the following
evidence:
i. Testimony by H that the defendants car was
not moving (dual purpose of providing
counter-evidence and impeaching plaintiffs
eyewitness).
ii. Testimony by I that plaintiff and eyewitness
had a year-long relationship prior to the
accident. (Dual purpose of showing bias and
contradicting eyewitness, showing the
eyewitness to be a liar).
iii. Testimony by J that drug store was closed that
day. (Does contradict but no other purpose so
the court may say it is collateral and exclude
under 403 & 611).
VI. Hearsay
a. Brief overview
i. The problem w/ hearsay is that you cant test the person who
heard/saw the original event (this person is the declarant).
ii. Rule of reliability: want to make sure evidence is truthful/reliable.
iii. Hearsay is ultimately about the reliability the jury hears.
iv. Witness angle
1. A witness sees the shooting (in her perception/memory) and
narrates this in court w/ sincerity. Then I saw . . .
2. The problem w/ hearsay is the declarant is narrating
someone elses point of view/sincerity.
b. Definition 801 (a)-(c)
i. Actual definition
1. Statement: a persons oral assertion, written assertion, or
nonverbal conduct if the person intended it as an
assertion.
a. Needs to be from a person
b. From an animal is not hearsay; machines, like
animals, are not hearsay
c. Emails, on the otherhand, would be hearsay.
2. Declarant: person who made the statement.
3. Hearsay means a statement that:
a. The declarant does not make while testifying at the
current trial or hearing; and
b. A party offers in evidence to prove the trust of the
matter asserted in the statement (by the declarant).
ii. Shorthand definition
1. Out of court statement offered to prove the truth of the
matter asserted in the statement, by the declarant.
2. What are the key issues?
a. Statement intended to assert something.
b. Statement was not made in the courtroom.
c. Being used at trial to prove a material fact of the
case.
d. Offered AT TRIAL to prove the trust of the matter
asserted IN THE STATEMENT.
3. General Rule: if its hearsay, and doesnt meet one of the
exceptions, it doesnt come in.
iii. Hypos:
1. In a criminal child abuse case a teacher testifies that the
alleged victim came up to her at recess and said
sometimes my step dad touches my pee pee.
a. Who is testifying in court?
i. teacher
b. Who is the declarant?
i. The child
c. What is the statement? (What is being asserted)
i. that the child inappropriately was touched by
her dad
d. What is it being offered in evidence to prove?
i. To prove he is a child molester.
e. Hearsay?
i. Yes, classic hearsay.
2. Criminal case: Cop testifies to a written statement of
Witness A who told the police that he saw man matching the
defendants description carrying a gun and money leaving
the bank.
a. Who is testifying in court?
i. The cop
b. Who is the declarant?
i. Witness A
c. What is the statement? (What is being asserted)
i. That a person matching the defendants
description, carrying a gun, left the bank w/
money.
d. What is it being offered in evidence to prove?
i. To prove guilt
e. Hearsay?
i. Yes, hearsay.
3. Bank Robbery: 3 eyewitness
a. Out of court statement by P: Defendant is the one
who did it. Is it hearsay?
i. Yes, hearsay
b. Out of court statement of S: The defendant went
out of here carrying money bags. Is it hearsay?
i. Yes, hearsay
c. Out of court statement of O: They ought to put
defendant in jail for this and throw away the key. Is
it hearsay?
i. Probably not hearsay because its an opinion,
but it could be hearsay (50-50)
iv. Two overriding questions
1. Is the litigant offering the statement to prove the truth of
what the statement says or was meant to say?
2. Did the declarant assert or mean to communicate that fact?
c. Nonhearsay Use of Out-of-Court Statements
i. Words offered to prove their effect on the listener
1. Language offered to prove the defendant had reason to fear
an attack.
2. Basically, language made for the truth of the matter
asserted.
ii. Legally operative words (verbal acts).
1. Such legally operative words are verbal acts because they
have legal force independent of the speakers intended
meaning (ex. I accept, I do).
2. These statements operate independently of the speakers
belief or intended meaning. The soundness of the
testimonial capacities therefore doesnt matter, so there is
no reason to exclude the words as hearsay.
3. Legally operative language is offered for what it does, not
what it says. It is deemed not to be offered for the truth of
the matter asserted because it is not offered as an
assertion. It does not matter that there is an overlap
between the content of the utterance and the matter sought
to be proven, so long as the utterance is legally operative
language.
iii. To impeach the declarants later, in-court testimony
d. Need to take into account FRE 613
i. Example:
1. Witness: blue car ran red light
2. Cop was told by witness: green car ran red light
a. Was the witness under oath, demeanor and cross at
this time? No.
b. If offered not for assertion, but to show
inconsistency, in witness court statement would not
be hearsay.
c. Could however be hearsay if used to show green car
ran the light.
i. Would need a FRE 105 limiting instruction to
show statement for impeachment and not
who ran the light.
ii. More or less used for the witnesss prior statement.
e. Book hypos
i. Problem 7.1, p.384
1. Yes hearsay just because its an affidavit doesnt mean its
not hearsay.
ii. Problem 7.2, p. 384-385
1. Yes, the gesture is hearsay; context is everything.
iii. Problem 7.3, p. 385
1. Yes hearsay at the time she made the statement, she was
not open to oath, demeanor, and cross.
iv. Problem 7.9, p. 391
1. Not hearsay he didnt intend it as an assertion; doing it
sincerely.
v. Problem 7.10, p. 392
1. Yes hearsay its an assertion because hed doing it for the
medias benefit.
f. The Nature of Assertions
i. There is a distinction between hearsay/non-hearsay.
ii. Conduct
1. Clearly assertive.
2. Sincerity is extremely important regarding assertions.
iii. Focus on Context
1. When trying to determine if something is an assertion, have
to ask could this conduct be a lie?
2. The burden is on the party claiming assertion.
3. Close cases should lean towards admissibility.
iv. Words
1. Most oral/written expressions are assertive Dont run that
stop sign
2. However, close the door is ambiguous
a. Depends on context.
v. Implied assertions
1. Intends to communicate those facts by implication.
2. No reason to distinguish sharply between express and
implied assertions.
a. Sometimes implied assertions is mistakenly used.
vi. Indirect assertions
1. Sometimes the matter asserted is just one link in a chain of
inferences leading to the ultimate fact to be proved.
2. Can use statements to prove something other than what it
asserts.
vii. Nonhearsay Use of Out-of-Court statements (398-99)
1. Nonassertive words
a. Ouch!
2. Words offered to prove something other than what they
assert
a. Sometimes, a lawyer offers the declarants words to
prove something other than what the declarant
intended to communicate. Here the lawyers claim
does not depend on the trust of the declarants
assertion.
3. Assertions offered as Circumstantial Proof of Knowledge
a. Close correspondence between what a description
was and was it actually was.
g. More hypos
i. Video clip hypo man charged w/ killing g/f; what witness heard
(about their marriage)
1. not hearsay because we dont care about the truth of the
statement, only that it was said (looking at effect on listener
establishes motive).
ii. Video clip - Woman murders her husband over threats
1. Victim is declarant
2. Victim asserting a threat
3. Declarant was at trial
4. Purpose? For self-defense
a. Being used for another purpose (a perceived threat)
being used for the effect on the listener (not
hearsay).
iii. Video clip direct testimony of witness in a sexual harassment suit
1. Statement? Yes, precautions were taken by female
employees.
2. Assertion? Yes, they were afraid of boss while he drank
because of sexual assault possibility
3. Declarant? Witness made it
4. At the time she made it wasnt under oath, demeanor, cross.
5. Purpose? Depends on context
a. If showing a hostile working environment, then yes,
hearsay (if suing him individually).
b. However, if offered to show company on notice, then
not hearsay (if suing company as a whole).
iv. Verbal acts hypo:
1. Government prosecuting a massage parlor owner for
soliciting sex. Undercover agent is called to testify about
conversation he had with woman giving a massage where
she asked whether I was interested in a good time and
the cost depends on what you want, but Im versatile and
you can have it any way you want it honey.
a. Statement? Yes, oral
b. Assertions? Yes
c. Declarant? The woman
d. At the time they were made, out of court.
e. Purpose? To prove statement was made, not
solicitation
i. Its a verbal act (not hearsay)
ii. Whats important here is it has different
purpose and independent significance
v. Anna killed by bus hypo
1. Anna was killed by a bus. Husband sues for wrongful death
and loss of companionship. To counter the claim that
Husband would have loss of companionship Bus company
wants to introduce evidence from Annas will executed a few
weeks before her death which said that Husband was cruel,
selfish and wasteful and she disinherited him, giving him
only a dollar.
a. Written, asserted, out of court statement by
declarant
b. Why is it being offered: to show there was already a
companionship lost.
vi. FBI agent hypo
1. FBI agents have the identity of H. as the perpetrator of a
bank robbery. They go to H.s house to arrest him. Wife
comes to the door and says H. is not here, he is in Denver
because his mother just died. He left on the 9th and wont
be home till the 15th. Police follow up on lead and
determine that wife is lying. They get information that H is
hiding out at his brothers house. They find him there. At
trial they want to introduce wifes statement to FBI agents.
a. Cop testifies about what wife said.
b. Truth of matter asserted? No, its a lie being
offered for a lie.
c. guilty knowledge she knew he robbed the bank.
d. Her lie established guilty knowledge.
e. NOT hearsay.
VII. Hearsay Exceptions
a. Overview
i. 5 categories of Hearsay Exceptions
1. Prior Statements by witnesses (801 (d)(1))
2. Statements by Party-Opponents (801(d)(2))
3. Declarants Availability Immaterial (803)
4. Declarant Unavailable (804)
5. Residual Exception (807)
ii. Most exceptions to the rule reflect two values: necessity and
trustworthiness
b. 801 (d)(2) An Opposing Partys Statement: The statement is offered
against an opposing party and:
i. (A) was made by the party in an individual or representative
capacity
1. Example from video: statement by made to a cop about a
crying baby.
a. This is classic hearsay, because its showing motive;
but, an exception applies.
2. Rationale
a. Its the adversarial environment; therefore it is fair
game as long as its relevant.
b. Any statements you make you have to live with.
c. Likely to be more truthful; wouldnt lie.
d. Some of the hearsay concerns dont apply.
e. Broad rule no personal knowledge of statements
made.
3. If he has a representative capacity and the statement is
offered against him in that capacity, no inquiry whether he
was acting in the representative capacity in making the
statement is required; the statement need only be relevant
to representative affairs.
4. Examples
a. Problem 7.11, page 410 Billables
i. Statement: she worked 104 hours
ii. Out of court statement: yes
iii. Why offering: wants to show worked 104
hrs.
iv. Truth of matter asserted: classic hearsay
v. However, fits w/ 801(d)(2)(A)
b. Problem 7.12, page 410-11 Take My Blood
i. Statement: take my blood.
ii. Out of court statement: yes
iii. Proof that it was said
iv. Why offering: if guilty, wouldnt offer blood
(proof of innocence)
v. Truth of matter asserted: yes, arguing had
nothing to hide (indirectly)
vi. Exception doesnt work under 801(d)(2)(A)
because offering it for his own purposes.
ii. (B) Adopted admissions/statements
1. Video clip: criminal case and on trial for insurance fraud
a. Ins owner statement admitted this by not denying
it
b. needs to hear statement
c. had chance to respond and didnt
d. Defense could argue s confusion over wood
shavings.
2. Doyle v. Ohio
a. After someone given Miranda, if they are still silent
after, cant be used against them.
3. Silence
a. Rationale
i. When silence is relied upon, the theory is that
the person would, under the circums, protest
the statement made in his presence, if untrue.
b. Civil cases: the results have been satisfactory.
c. Criminal cases: troublesome questions by failing to
deny an admission; however, the rule contains no
special provisions concerning failure to deny in
criminal cases.
4. Requirements
a. That the party against whom the statement is being
offered heard the statement.
b. That the party could have responded.
c. That the circumstances naturally called for a
response.
d. The party failed to respond or deny (or responded
but did not rebut)
iii. (C) Speaking Agents
1. ***The statement must be considered but does not by itself
establish the declarants authority.***
2. Broad based rule.
3. Communications to an outsider hasnt generally been
thought to be an essential characteristic of an admission.
4. Examples: atty; a partys books or records are usable
against him, w/o regard to any intent to disclose to 3rd
persons.
iv. (D) was made by the partys agent or employee on a matter within
the scope of that relationship and while it existed.
1. ***The statement must be considered but does not by itself
establish the existence or scope of the relationship.***
2. Usually, based on the basis of an employment relationship.
3. Can be both broader and narrower than 801 (d)(2)(C)
4. Mahlandt v. Wild Candid Survival
a. Facts: Daniel going to get his brother at a neighbors
house; wolf allegedly attacked boy inside the wolfs
case; wolf previously attacked beagle; Mr. Poos had
the wolf to show they werent dangerous to humans.
Plaintiff going after Mr. Poos company (assuming
plaintiff going after Poos too). Mr Poos 1. Writes a
note; 2. Tells Mr. Sexton Sophie bit a child; 3. Board
meeting notes.
b. Issue: Whether or not these statements are
admissible?
c. Holding:
i. 1. Hearsay, but gets in under 801 (d)(2)(A)
1. This is regardless if he actually knew,
because here he didnt know if Sophie
actually bit boy.
ii. 2. Hearsay, and again for the same reason as
1.
1. Want to use these statements against
the company (worked for Mr. Poos)
2. Should be allowed in (Mr. Poos and co.
said it):
a. He was an agent of the
company
b. Being used against the
company
c. w/in the scope of the
relationship/employment while
it existed Mr. Poos had control
over the wolf.
3. The company, in essence, says it when
Mr. Poos says it.
iii. Board meeting notes
1. Would be admissible under (d)(2)(A) or
(C) against the company.
2. Wouldnt work for Mr. Poos though
(company isnt an agent for him.
5. Problem 7.15, p. 419 Shovel and Bucket
a. Clearly hearsay
b. How to prove (judge decides under 104):
i. Employee relationship exists: showed up w/
shovel after phone call; knowledge of
schedule (can be used w/in context)
c. How to prove w/in the scope of employment:
i. Against statement: not his job to comment on
schedule
ii. For statement: his job contingent on their job.
6. Hypos
a. Security guard who was involved in the shooting and
said we went a little crazy.
i. Yes, under 801 (d)(2)(D)
ii. Need something more too
1. Contract would be good evidence;
paycheck; past performance; uniform
2. Statement alone isnt enough
b. Other Blackwater employee who investigated the
shooting incident and told his boss, They were
poorly trained.
i. Yes, based on scope of investigation under
801 (d)(2)(D)
ii. Hired to do this assessment
iii. If he was told not to say anything, and keep it
privileged, it would still be admissible.
c. Another Blackwater employee (truck driver) who was
driving by on an unrelated mission and says No one
was shooting at them.
i. Depends on whether it can be used.
ii. Could be outside of relationship; but may
need more information.
v. (E) Co-conspiracy
1. ***The statement must be considered but does not by itself
establish the existence of the conspiracy or participation in
it.***
2. Elements of Co-Conspirator Statements
a. Must prove that the parties conspired i.e. the
existence of the conspiracy (Coventurer requirement)
b. Statement made during the conspiracy (Pendency
requirement)
c. Made in furtherance of the conspiracy (Furtherance
requirement)
3. The Supreme Court ruled that these 3 elements were to be
decided by a preponderance of the evidence.
a. ***of course, this could be different since the
remodeled rule***
4. Bourjaily v. U.S.
a. Holding: You can use statements; but the statement
themselves are not enough.
5. ***The limitation upon the admissibility of statements of
coconspirators to those made during the course and in
furtherance of the conspiracy is in the accepted pattern.
vi. FRE 805
1. Hearsay w/in hearsay is not excluded by the rule against
hearsay if each part of the combined statements conforms
w/ an exception to the rule.
2. Hypo
a. Cop testifies in state court about a conversation
between D and L that L told cop.
b. This is where multiple levels of hearsay lie no
exception for this; doesnt get in.
c. REMEMBER, follow the bouncing ball.
c. 801 (d)(1) Prior Statements by Witnesses
i. Preliminary Requirements
1. The declarant testifies at the present trial or hearing and is
now subject to cross-examination concerning the prior out
of court statement, and the prior out of court statement is . .
.
a. Declarant must be testifying in court now about a
statement they made out of court.
b. Declarant must now be subject to cross-examination
about the out of court statement.
2. *** you could just use FRE 613 to impeach***
a. However, if you want to go to the truth of the matter
asserted, you go to 801 (d)(1)(A).
ii. Then, the statements must meet one of the following conditions
1. 801 (d)(1)(A) inconsistent statements
a. Language: Inconsistent w/ the declarants testimony
and was given under the penalty of perjury at a trial,
hearing, or other proceeding or in a deposition;
b. Very narrow exception
i. The other proceeding is a grand jury
proceeding defendants attorneys would not
be allowed in but could be; these are typically
prosecutor run.
c. Good overview: The declarant must testify at current
proceeding and be subject to cross-examination
about the prior statement AND the past statement
is inconsistent and was given under oath at a
proceeding or deposition.
d. Conditions
i. Inconsistent w/ the in court testimony (need
this to trigger impeachment)
ii. Out of court statement was given under oath
subject to the penalty of perjury at a prior
trail, prior hearing, other proceeding,
deposition
1. ***no requirement for prior cross-
examination***
e. Distinction btwn 801 (d)(1)(A) and FRE 613
i. A jury often may not grasp the distinction
between using an out-of-court statement
substantively and using it only to impeach.
This problem does not arise when a witnesss
past inconsistent statement is admitted under
this rule. Such statements come in
substantively, and the jury may consider
them for their truth. Far more stringent
conditions need to be met to admit a past
inconsistent statement for its truth under this
rule than to merely impeach under FRE 613.
f. Problem 7.17, page 438-439 Retraction
i. Brenda Raymond is the declarant.
ii. The statement is not getting in and the judge
should direct a verdict for acquittal.
g. Problem 7.19, page 453 Domestic Violence I
i. Yes, per se of the rule.
2. 801 (d)(1)(B) consistent statements
a. Language: is consistent w/ the declarants testimony
and is offered to rebut an express or implied charge
that the declarant recently fabricated it or acted from
a recent improper influence or motive in so
testifying.
b. Conditions
i. The statement is consistent w/ prior testimony
ii. Rebuts an express or implied charge of recent
fabrication, improper influence, improper
motive
1. ***purpose is to rebut, not bolster***
c. Timing of when motive established is the critical
issue here
i. Important to know when motive came into
play.
ii. For instance, if motive was established before
the prior statement, it would be problematic.
However, if after the prior statement, better
chance to get in.
iii. ______________________________________________
____
Motive (problematic) prior stmt motive
ct. stmt
iv. Court hasnt addressed whether all motives
need to be rebutted; courts have said just a
motive enough.
v. Tome v. U.S.
1. Facts: child sexual abuse case, during
cross, D raised the issue that the child
was motivated by her desire to live
with her mother. Government wanted
to offer testimony by 6 witnesses
describing statements that the child
made to disprove this issue.
2. Rule: NOT admissible b/c the prior
consistent statement MUST come
BEFORE the motive that she can live
with her mother. The government was
unable to pinpoint when that motive
arose, therefore the statements were
NOT used to rebut a charge of
improper motive. Tome is binding
authority that the prior statement has
to come before the improper motive or
influence. NOT required by the rules,
but it is NOT a requirement.
3. 801 (d)(1)(C)
a. Language: identifies a person as someone the
declarant perceived earlier
b. What the rule does
i. It allows the declarant to refer back to a prior
statement
1. Out of court statement would be more
reliable than an in court statement.
c. Video clip we want the statement thats him in
i. Prosecution questions: were you wearing
glasses both times (line-up and at crime);
seen him before?
ii. Go through time at the police station until
making statement
iii. Out of court statement more reliable; after
declarant testifies, cop could always testify
too.
iv. Could STILL bring in the statement even if the
declarant doesnt remember.
d. Rule 804 Declarants Unavailability (restricted)
i. 804 (a) Criteria for unavailability
1. ***it doesnt apply if the statements proponent procured or
wrongfully caused the declarants unavailability as a witness
in order to prevent the declarant from attending or
testifying***
a. (1) exemption (court needs to find/rule that privilege
applies)
i. Self-incrimination; spousal; atty-client
ii. Need to go to state to see specific privileges
iii. Witness usually has to go to the stand to
assert except a when asserting their 5th
Amendment right.
b. (2) refuses to testify despite a court order
c. (3) testifies as not remembering on the subject
matter
i. Specifically, the substance
d. (4) because of death, mental illness, some other
physical or mental infirmity
i. Fact dependent
e. (5) absent and cant be found
i. Need to show exhaustive legal efforts made
2. The very first question under an 804 analysis is
whether the witness is available. If the witness is
available, then get out of here!!!!!!!
3. Problem 7.22, page 474-475 Domestic Violence IV
a. She is unavailable (804 (a)(2))
b. However, at the grand jury there was no opportunity
to cross.
ii. 804 (b) Exceptions: REMEMBER, declarant has to be unavailable
1. 1. Former Testimony narrow exception
a. Elements
i. Testimony given at former hearing
ii. Under oath
iii. Party against whom the statement is now
being offered had an opportunity to question
the declarant at that hearing
iv. Substantial identity of issue between the 2
hearings
v. Substantial identity of parties
b. Problem 7.23, page 475
i. Unavailable? Yes
ii. What would you want to know?
1. Motive for defense; difference between
civil/criminal proceeding set-up.
2. Remember, need to have an
opportunity to cross and similar
motive.
c. If witness says one thing at 1st trial, want to bring in
the court document/transcript
i. Dealing w/ 2 levels of hearsay: the prior
statement and the court document/transcript.
2. 2. Statement Under the Belief of Imminent Death
a. Elements
i. Criminal homicide case or civil case
ii. Declarant is the victim
iii. Declarant has belief in impending death
iv. Statement relates to the cause or
circumstances of the believed impending
death.
b. Death must be imminent: i.e., looming, threatening,
menacing
c. Video clip: wrote Doyle before he died
i. Is declarant unavailable: yes, hes dead
ii. Statement oral or written? Yes, written
iii. Imminent: here, yes
d. One thing w/ this rule, death doesnt have to result;
just need to believe that death is imminent when the
statement was made
e. Hypo1
i. Homicide prosecution, declarant staggers into
ER, husband just beat me, dies 3-4 hours
later.
1. Unavailable there
2. However, imminency depends on the
facts and circumstances
f. Hypo2
i. Same facts as hypo 1, but instead the
husband stole $20k?
1. The $20k doesnt correlate to the
death.
g. Hypo 3
i. Same hypo, but instead the declarant says if
my husband doesnt stop Ill die.
1. No imminency.
h. Shepard v. U.S.
i. Facts: Wife, prior to her death, says that
defendant poisoned her. She made this
statement a few weeks before death; at times
seemed to be getting better.
ii. Issue: whether or not the conversation can be
admitted?
iii. Holding: the statement cannot be used
because imminency not there; has to be fairly
close imminency between fear and death.
3. 3. Statement Against Interest
a. Easy rule to confuse w/ 801(d)(1)(A) KEEP IT
SEPARATE
b. ***appears to be a very limited rule***
i. The rule is predicated on its rationale: a
reasonable person wouldnt say something to
implicate themselves if it wasnt true.
ii. Context is everything
c. Elements
i. Declarant believed that the statement was
against his interests
ii. The interest is one covered in the rule
(pecuniary, property, penal, civil liberty)
iii. Declarant unavailable at trial
iv. It is offered in a criminal case as one that
tends to expose the declarant to criminal
liability there must be corroborating evidence.
d. Williamson v. U.S.
i. ***keep in mind, a potential confrontation
clause issue here as well***
ii. Facts: Harris pulled over; refuses to testify w/
respect to Williamsons story. Harris gave true
story eventually and directly implicates
Williamson. Harris, in essence, saying he was
guilty of transporting cocaine.
1. His primary motivation was to shift
burden.
2. Seems like 804(b)(3) but Harris is just
minimizing fault.
iii. Holding: the court wants to look at the
statements, line by line, to see if they are
inculpatory versus Williamson or neutral
1. The neutral ones allowed in.
2. This is a very narrow view.
4. 6. Statement Offered Against a Party That Wrongfully
Caused the Declarants Unavailability
a. Elements
i. The defendant engaged or acquiesced in the
wrongdoing
ii. The wrongdoing was intended to render the
declarant unavailable as a witness
iii. The wrongdoing did cause the witness to be
unavailable
b. ***does not need to be criminal conduct***
c. Hypo
i. Two co-defendants are charged w/ robbing a
convenience store. D1 decides to come clean
and tells the cops about their involvement
and said that D2 fired the shots. D1 pleaded
guilty and agrees to testify against D2.
However, before trail, D1 gets a threatening
letter from D2 saying that if he testifies, he
and his g/f will be hurt/killed. Judge conducts
a preliminary hearing and determined that D2
gave the threat.
1. Statement conforms w/ the rule.
2. Rationale: cant shield yourself w/
hearsay rule
d. U.S. v. Gray
i. Facts: Crazy chick who kills husbands for
insurance money; insurance company wants
to use former husband (Grays) statements.
ii. Issue: Whether or not the defendant made
Gray unavailable for this particular trial?
iii. Holding: The court doesnt limit to a
particular trial; just a trial in general.
e. 803 Declarants Unavailability Immaterial - Unrestricted
i. Overview of the rules
1. These are unrestricted hearsay exception because it doesnt
matter if the declarant is available to testify, the out of court
statements can come in for their substance, in most cases
w/o the declarant taking the stand.
2. How do the exceptions differ?
a. 801(d)(1) exceptions requires the declarant to testify.
Exception is based on delayed cross-examination.
b. 801 (d)(2) exception based on the status of the
declarant and his/her relationship to the case.
c. 804 exceptions based on a showing of declarant
unavailability and are more limited.
d. 803 exceptions unrestricted based on belief that the
out of court statement has sufficient guarantees of
reliability.
3. 3 mechanisms for testing hearsay: oath, demeanor, cross-
examination
a. 803 exceptions make the chance of fabrication
unlikely
b. Subjecting these to the 3 mechanisms arent going to
add to truthfulness
c. Theyre as good as having the declarant on the
stand.
d. Better to get hearsay into evidence than the
declarants testimony
4. When thinking of these rules, more than one can apply.
ii. 803(1) Present Sense Impression
1. A statement describing or explaining an event or condition,
made while or immediately after the declarant perceived it.
2. Elements
a. An event occurred
b. Declarant has personal knowledge of the event
c. Declarant made the statement while perceiving the
event or immediately thereafter
d. Statement describes or explains the event.
3. Example
a. Willie Mays catch described the event while
declarant perceived it
b. ***the announcer stated what he saw***
iii. 803(2) Excited Utterance
1. A statement relating to a startling event or condition, made
while the declarant was under the stress of excitement that
it caused.
2. Elements
a. Startling or stressful event occurred
b. Declarant has personal knowledge of the event
i. This can be proved circumstantially
c. Declarant made a statement relating to the event
i. Broader in scope then present sense
impression
d. Declarant made the statement while still under the
stress or excitement of the event.
i. This is the key and while immediacy is still a
factor, it is not dispositive.
3. Problem 7.29, page 512 Dog Mauling I
a. dog just bit me
i. Should be able to get in under this rule
because would still be under the stress.
Again, its very contextual.
4. Problem 7.30, page 513 Dog Mauling II
a. Should be able to get in under both 803(1) and 803
(2).
5. Problem 7.31, page 513-514 Recantation
a. Statement could be used to impeach w/ a prior,
inconsistent statement (FRE 612).
i. But this doesnt prove the truth of the matter
asserted.
b. Could probably be brought in under 803(2) w/
supporting testimony from the officer.
iv. 803(3) Then-Existing Mental, Emotional, or Physical Condition
1. Elements
a. To prove then existing physical/mental/emotional
condition
b. To prove later conduct
c. To prove facts about a will
d. NOT to prove fact remembered or believed (except
for will)
2. Rationale: nobodys a better source of how you feel than
you are.
a. Could make the argument, depending on the
circumstances, that this isnt even hearsay.
b. You cant infer someone elses plan; focus is on the
declarant.
i. Another question is how contemporaneous
does it need to be?
c. Need to be careful not to confuse w/ 803(4) doesnt
need to be said to a doctor.
d. The rule is NOT background looking (video clip
example)
i. I meant to do that saying it before the fact
is fine; after an issue.
ii. I will have to disappear if pregnant w/
someone elses baby statement of a plan;
fits under 803(3).
3. Mutual Life Insurance Company v. Hillmon
a. Facts: Hillmon allegedly dead; wife bringing claims
to insurance company; insurance company saying
its fraud and they believe Walters is dead; 2 letters
written by Walters: one telling his sister that he
intended to leave Wichita on March 5th with Mr.
Hillmon for Colorado or parts unknown to me. In the
second letter, he told his fianc that he intended to
leave Wichita to see a part of the country I never
expected to see and I am going with a man named
Hillmon who intends to start a sheep ranch. Letters
were excluded from trial.
b. Issue: whether or not the letters can come in?
c. Holding: Court allowed it all in, but there is a
hearsay problem w/ whether Hillmon planned on
being there. However, court said a new trial was
needed that included the letters.
d. Rule: letters admissible, they are a forward looking
statement of his future intent. NOT to infer that the
two men met up with each other, that is backward
looking. Have to be careful about inferring conduct
of others from statement used to show conduct of
declarant.
4. Problem 7.33, page 519
a. (a) yes, intent to act in a certain way (buy marijuana
and come back)
b. (b) to prove Angelo kidnapped? going to prove
something more, would be hearsay under 803(3);
would be allowed in under Hillmon rule.
v. 803(4) Statement Made for Medical Diagnosis or Treatment
1. Elements
a. Statement made to the proper addressee.
b. Declarant knew that the person was a proper
addressee.
c. Declarant made the statement with a medical motive
(treatment/diagnosis).
d. Declarant realized that statement was medically
relevant.
2. Focus is on the declarant, but problematic if the declarant
not there.
3. In these instances, the injured party is the declarant.
a. The declarant has no reason to lie and wants to take
the pain/injury away.
4. Problem 7.34, Page 531-532 Elder Abuse
a. Convo w/ lawyer wouldnt come in under 803(4).
b. Statement w/ doctor about headaches/nausea should
get in.
c. How he fell also should get in because its medically
relevant.
d. w/ respect to identification of the pusher
i. defense would say not relevant because test
would be the same.
ii. For prosecutor, would look to prevent future
incidents.
5. Legal components typically arent medically relevant.
a. Statements to fault usually do not qualify. For
example, a patients statement that he was struck by
an automobile would qualify but not his statement
that the car was driven through a red light. Another
example concludes that a statement by a patient
that he was shot would be admissible but a
statement that he was shot by a white man would
not be. And the fact that a patient strained himself
while operating a machine may be significant to
treatment but the fact that the patient said the
machine was defective may not be.
6. U.S. v. Iron Shell
a. Facts: 9 year old girl allegedly assaulting by Iron; the
key questions at trail concerned the nature of the
assault; Doctors statements, on the night of the
incident w/ the girl explaining what happened, were
allowed in at the lower court and defendant
convicted.
b. Test from Iron Shell
i. Is the declarants motive consistent w/ the
purpose of the rule?
ii. Is it reasonable for the physician to rely on the
information in diagnosis or treatment?
c. The statements made were allowed in.
vi. 803 (5) Recorded Recollections
1. Elements
a. The witness lacks present recollection.
i. Note 612 and the rule regarding attempts to
refresh.
b. The recording (statement) accurately reflects
knowledge the witness once had.
c. The witness made or adopted the earlier statement.
d. The recording was made while the matter was fresh
in his mind.
2. SOMEWHAT predicated for declarant being at trial.
3. HAS TO BE A WRITING
4. ***IF ADMITTED, THE RECORD MAY BE READ INTO EVIDENCE
BUT MAY BE RECEIVED AS AN EXHIBIT ONLY IF OFERED BY
AN ADVERSE PARTY***
5. Problem 7.38, page 542 License Plate
a. M yells number; S writes it down
b. Concerns:
i. S doesnt have personal knowledge hearsay
issue would be there.
ii. M could get it under 803(1)
iii. Then, would get envelope and give it to S
c. Want to establish that at the time you wrote it, it was
accurate.
d. HAS to be a writing
vii. 803(6) Records of a Regularly Conducted Activity
1. Elements
a. Regular business and regularly kept record
b. The source of the information must have personal
knowledge of the information
c. Information gathered/recorded must be close in time
of the event
d. Records custodian or self-authenticating document
must establish information about the record keeping
system, but does not need personal knowledge of
the information recorded.
2. Hypo: I take train in everyday; get parking ticket one day;
dont pay; parking lot owner take me to court.
a. w/o 803(6), would put a burden on businesses
3. purpose CANNOT be for litigation; used for regular business
purposes
4. business has a motivation to be truthful and accurate
5. How routine is routine?
a. The person present at trial doesnt need to have
personal knowledge; just knowledge of the PROCESS.
6. Video: medical malpractice; negligence w/ anesthesia
a. The info about when she ate would get in under
803(4)
b. Assuming the nurse isnt there, should be able to get
in under 803(6) would need to call another
nurse/tech.
7. Problem 7.39, page 555-556
a. (1) the record would not be able to get in under
803(6)
i. Person who has direct knowledge is not part
of organization because customer not part of
organization.
b. (2) could get in under 801 (d)(2)
i. Broad rule but need to be part of the
organization.
8. Can be a very broad rule: can be a charity or a non-profit
a. Rationale is because they want truthful and accurate
information.
9. Record seen as more reliable than the declarant.
viii. 803(8) Public Records
1. Elements
a. Record Types Allowed Under the Rule
i. Activities of the office
ii. Matters observed pursuant to a legal duty
(but not law enforcement reports)
iii. Factual findings resulting from investigations
pursuant to legal authority
b. Recorded in official custody
c. Record was properly prepared
d. Preparer was a public official
e. Official had a duty to record the information
f. Official had personal knowledge (except in cases
under 803(8)(iii) in a civil case or against the govt. in
a criminal case, factual findings from a legally
authorized investigation)
g. Entry was factual in nature
2. Distinction between this and 803(6) is govt. organizations
keeping records here as opposed to a private entity
3. Tricky thing:
a. One type doesnt work: law enforcement records
against a in a criminal case (***confrontation
clause problem***)
b. Many states also dont have the 803(8)(A)(ii)
exception
4. Overall, a lot of statements come in.
5. Public record is an easier foundation to make govt. has a
duty
a. The difference is, do they have a duty to report?
6. Record seen as more reliable than the declarant.
7. ***courts are split on whether a denial under 803(8) could
be let in under 803(6)***
ix. 803(7) Absence of a Record of a Regularly Conducted Authority
1. Elements (all 3 must be present)
a. Didnt exist or occur
b. Record regularly kept
c. No lack of trustworthiness
f. 807 Residual Exception
i. Elements
1. The statement is offered as evidence of a material fact.
2. The statement is more probative on the point for which it is
offered than any other evidence which the proponent can
procure through reasonable efforts.
3. The general purposes of these rules and the interests of
justice will best be served by admission of the statement
into evidence.
4. Notice
a. Reasonable notice also required w/ additional
information.
ii. Rule used frequently in child abuse cases
1. If you cant find any exceptions under 803 or 804, got to 807
and try to get it in.
2. Video of quasi-social worker w/ child; Hypo: and at trial,
child isnt going to testify; would any of exceptions fit?
a. Probably the best one would be 803(4) but that
depends on the circumstances.
b. Wouldnt apply to business records 803(6)
c. The best would be 807
3. near miss exception
4. Idaho v. Wright
a. Court says you cant use extrinsic evidence to exert
trustworthiness
g. 805 Hearsay w/in Hearsay
i. Hearsay w/in hearsay in not excluded by the rule against hearsay if
each part of the combined statements confirms w/ an exception to
the rule.
h. 806 Attacking and Supporting the Declarants Credibility
i. Hearsay statement, or a statement in 801 (d)(2)(C), (D), or (E), has
been admitted into evidence, the declarants credibility may be
attacked, and the supported by any evidence that would be
admissible for those purposes if the declarant testified as a witness.
i. Confrontation Clause 6th Amendment
i. Language
1. In all criminal prosecutions, the accused shall enjoy the right
. . . to be confronted w/ the witnesses against him . . .
ii. Basic terms of the Confrontation Clause
1. Applies in criminal prosecutions ONLY
2. Applies only to the rights of the defendant; prosecution
doesnt get this right
3. Confrontation in some form is required
4. Cross-examination: the confrontation is to cross-examine
iii. If read broadly . . .
1. There could be no hearsay exceptions
2. Rule will allow some hearsay while others not allowing it in.
iv. Additional things to remember
1. Once you go through the hearsay analysis, see if exception
applies; if an exception applies, see if there is a
confrontation clause problem.
2. Nothing is automatic just because you satisfy hearsay
exception, doesnt mean you satisfy confrontation clause
3. Confrontation clause applies in criminal cases; Congress or
states can abolish or modify hearsay rules but not the
Clause itself.
4. Both the Confrontation Clause and the hearsay rules are
rules of exclusion. Each trumps the other. Confrontation
Clause, if it applies, will exclude hearsay evidence that
otherwise meets an exception. Hearsay rules, if they apply,
will exclude evidence that satisfies the Clause, if no
exception to the hearsay rules applies.
5. The scope of the Clause similar to the scope of the hearsay
rules because both have concerns about the inability to
cross-examine declarant.
v. Mattox v. U.S.
1. Facts: Prosecution wants to admit testimony of witness
where witnesses died after the first trial.
2. Holding: The court allows the statements in.
vi. Ohio v. Roberts
1. This case is no longer the law.DONT FOLLOW IT ON THE
EXAM!!!!!!
2. The court set out a two part interpretation
a. Necessity: whether or not the out-of-court statement
was subject to cross-examination, the prosecution
must either produce, or demonstrate the
unavailability of, the declarant whose statement it
wishes to use against the defendant.
b. Reliability (this was the biggest issue here): If the
declarant is unavailable, the hearsay statement is
admissible only if it bears adequate indicia or
reliability. Reliability can be inferred w/o more in a
case where the evidence falls w/in a firmly rooted
hearsay exception [or where the defendant had an
earlier, adequate chance to cross-examine the
declarant about the statement]. In other cases, the
evidence must be excluded, at least absent a
showing of particularized guarantees of
trustworthiness.
vii. Crawford v. Washington
1. Holding: The Confrontation Clause bars admission of
testimonial statements of a witness who did not appear at
trial unless he was unavailable to testify, and the
defendant had a prior opportunity for cross-examination.
(***very literal approach by Scalia***)
a. The focus is on testimonial statements.
b. Also, has to be a trial-like setting.
i. Would work in a prior trial
ii. MAYBE a pre-trial hearing
viii. Crawfords Testimonial Formulation
1. Testimony or function equivalent (affidavits, custodial
interrogations, prior un-cross-examined testimony) . . . that
declarant would reasonably expect to be used
prosecutorially.
2. Extrajudicial statements contained in formalized testimonial
materials (affidavits, depositions, prior testimony,
confessions) (Justice Thomass definition in White v. Illinois,
502 U.S. 346 (1992)).
3. Statements made under circumstances which would lead an
objective witness to reasonably believe statement would be
preserved for trial.
ix. Question left open after Crawford, what is testimonial?
1. Solemn declaration[s] or affirmation made for the purpose
of establishing or proving some fact. Statement made in
response to police interrogation, even if not sworn Note:
need not be sworn
2. Statements produced w/ the involvement of government
officers . . . w/ an eye toward trial
3. Prior testimony at a preliminary hearing, grand jury, former
trial
4. Non-testimonial: business records; dying declarations; on-
going emergencies; overheard remark.
x. What is not Testimonial?
1. An off-hand, overheard remark
2. casual remark to an acquaintance
3. statements in furtherance of a conspiracy:
4. At least some business records
xi. Gray Areas
1. Present sense impression/911 calls
2. Excited utterance
3. Medical diagnosis
4. Statements of children to social workers
5. Lab reports/business records
6. Forfeiture by wrongdoing (Waiver)
7. Statements against interest that implicate a co-defendant
xii. What happened to reliability?
1. The [Confrontation] Clauses ultimate goal is to ensure
reliability of evidence, but it is a procedural rather than a
substantive guarantee. It commands, not that evidence be
reliable, but that reliability be assessed in a particular
manner: by testing in the crucible of cross-examination.
xiii. Primary Purpose Test (from Davis/Hammon)
1. Statements are non-testimonial when made in the course of
police interrogation under circumstances objectively
indicating that the primary purpose of the interrogation is to
enable police assistance to meet an ongoing emergency.
2. They are testimonial when the circumstances objectively
indicate that there is no such ongoing emergency, and that
the primary purpose of the interrogation is to establish or
prove past events potentially relevant to later criminal
prosecution.
3. Factors:
a. Scope of the emergency
b. Type of dispute
c. Facts known to the police at the time of investigation
d. Potential threat to public
e. Whether suspect was apprehended
f. Medical condition of the declarant
g. Types of questions asked by the police (initial or
follow-up?)
h. Informality of the questioning, etc.
i. Taken in calm circumstances when the declarant out
of danger
j. Taken in the stationhouse or at the scene
k. Involving a series of questions
l. Being recorded by police interrogator
m. Following custody and Miranda warnings
n. Involving separation of the declarant from the
suspect
o. Being made in circumstances which deliberate
falsehood risked severe consequences
4. Video clip w/ child testimony: assume hearsay exception
applies is there a confrontation clause problem?
a. Defendant argues testimonial: its an interrogation;
referring to past events; werent spontaneous
questions; her purpose was to get statements for
trial.
b. Prosecution argues non-testimonial: declarant didnt
know purpose; on-going emergency (pending
circumstances); forfeiture of conf. cl. right (if purpose
to prevent witness testimony.
xiv. The point of all of the cases here is that the rule is really mushy.
xv. Michigan v. Bryant
1. Facts: Victim found dying in a gas station lot; shot earlier at
a drug dealers house down the road; cops ask a bunch of
questions; victim dies later at the emergency room.
Statements allowed in.
2. An ongoing emergency is only one (albeit very important)
factor in determining the primary purpose of an
interrogation. Meaning: look at other factors, including the
informality of the encounter.
3. In making the primary purpose determination, standard
rules of hearsay, designed to identify some statements as
reliable, will be relevant. Note how this seems to bring
back the reliability analysis and links hearsay exceptions to
a satisfaction of the CC requirements.
4. Primary purpose is an objective evaluation so we look at
both the declarants actions and the cops actions to make
that objective determination. Why is the cops purpose
relevant to how it would be perceived by the declarant?
5. Court said that the mixed motives also have to be taken into
account and a focus on objective facts surrounding the
taking of the statement will help sort out the mixed motives.
6. Can look through 2 sets of eyes: can look through
declarants eyes or the person taking the information down.
a. Scalia said only declrants view matters.
b. Majority wants to look at both.
xvi. Post-Crawford
1. Issues arose w/ respect to domestic violence
2. Issues also w/ written reports in prosecution would have to
bring in everyone on chain of evidence (plus the person
doing the testing)
xvii. Melendez-Diaz v. Mass. (2009)
1. Certain types of statements are sufficiently formal that, by
any account, they are testimonial (such as a forensic lab
report meant to establish the defendants BAC or to
determine whether a substance is cocaine).
xviii. Bullcoming v. New Mexico
1. FN6: a statement is testimonial if its primary purpose is to
establish or prove past event potentially relevant to later
criminal prosecutions.
2. Is FN 6 the law? Good argument that it is because Justice
Sotomayor, who wrote Bryant, agreed w/ it. BUT, only time
will tell.
xix. 4 justices (maybe 5?) ready to jettison Crawford revolution and
return to the days where the essential question was the reliability
of the statement, in other words, Ohio v. Roberts.
1. For now, Crawford, modified by Davis/Hammon, Bryant, and
Bullcoming is still good law. But it seems to be hanging on
by a thread.
xx. Key Points
1. When the declarant is present, testifies at trial, and
responds to questions about the out-of-court statement
there is no Confrontation Clause violation.
2. If, despite best efforts, the prosecutor cannot produce the
declarant, no Confrontation Clause problem when the out of
court statement was made under oath and subject to cross-
examination.
3. Also, no confrontation clause problem when the out of court
statement is not offered for the truth of the matter asserted.
xxi. Confrontation Clause on the Exam
1. If the question addresses a settled issue (i.e., clear ongoing
emergency, former testimony, an off-hand remark, co-
conspirator statement, etc.), you should state the law, apply
it, and move on.
2. If the question addresses a more ambiguous question (i.e.,
an ambiguous 911 call, child witness who tells a state social
worker about past sexual abuse, other), you need to argue
from the available factors.
3. Credit will be given for the quality of your arguments, not
merely your conclusion. Which factors suggest that a
statement is testimonial? Which do not? Why?
4. Williams v. IL: Dont have to worry about too much because
it was a judge alone case, not a jury trial; court found the
situation OK.
j. The Bruton Doctrine
i. (if applicable) the third question after hearsay and Confrontation
Clause.
ii. Concerns an out-of-court admission made by an accomplice tried
jointly w/ the defendant.
1. The confessing accomplices words are admissible against
him/her as an opposing partys statement; however if
offered against the defendant, they most likely would fail
Confrontation Clause scrutiny.
2. Situation arises when A says nothing and B implicates A.
iii. Problem as using it against A: no cross-examination (if A
unavailable); definitely a hearsay problem and maybe even a
confrontation clause issue.
iv. Ways to avoid confrontation violation (Bruton applies if none of
these opts avail):
1. Severed trials
2. Separate juries
3. Testimony by the confessing accomplice
4. Redaction
5. Bench trial
6. Admissibility of statement against non-maker
v. Bruton v. U.S.
1. Facts: a jury instruction given limiting the postal inspectors
testimony; Bruton convicted and appeals.
2. Issue: Whether the conviction of a defendant at a joint trial
should be set aside although the jury was instructed that a
codefendants confession inculpating the defendant had to
be disregarded in determining his guilt or innocence?
3. Holding: The Court holds because of the substantial risk
that the jury, despite instructions to the contrary, looked to
the incriminating extrajudicial statements in determining
petitioners guilty, admission of Evans confession in this
joint trial violated petitioners right of cross-examination
secured by the Confrontation Clause of the Sixth Amend.
vi. The point of Bruton is to show that a jury instruction limiting
testimony doesnt work.
VIII. Lay Witnesses and Experts
a. FRE 701: Opinion Testimony by Lay Witnesses
i. Overview
1. Lay witnesses: may not be in a position to give opinions
(i.e. negligent; tire failing)
2. In some form or another, theyre all opinions/legal
conclusions
3. Up to this point, we talked about the declarant having
personal knowledge; here, may not be direct observation.
ii. Opinion is limited to one that is (rule language):
1. (a) rationally based perception,
2. (b) helpful to understand testimony or helpful in determining
an issue of fact,
3. AND (c) not based on scientific, technical, or other
specialized knowledge in the scope of FRE 702.
iii. This person is NOT testifying as an expert.
iv. Examples:
1. Witness testifying to someone elses voice: personal
knowledge important
2. Handwriting: personal knowledge important
3. Blood type: depends on circumstances; pretty close to the
line
4. Drunkenness: should work
5. More prototypical examples: conduct, sound, weight
v. A lay witness saying I think theyre being truthful/lying is
problematic!
vi. In this type of questioning, a foundation needs to be laid first.
vii. Testimony is limited to what they observe.
b. Expert witness overview:
i. May be authority in something to give an opinion
ii. Why can expert opinions be helpful: pretty powerful evidence;
theres a lot at stake; however, there are many problems (such as
the inexactness of fingerprints)
iii. Can give a wider girth as to what they can testify to can become
one by time in field, degree, etc.; can be powerful
c. FRE 702 Testimony by Expert Witnesses
i. Individual has expertise and can testify if
1. (a) will be helpful to the trier of fact to understand evidence
or determine an issue of fact
2. (b) testimony is based on sufficient facts/data
3. (c) testimony is product of reliable principles/methods
4. AND (d) expert reliably applied principles/methods to the
cases facts
ii. Factors to be used
1. Qualified expert: training, skill, knowledge, experience;
quite broad; nexus needed between expert and evidence
2. Proper subject matter: stuff the jury may not know; in an
area where jury experience may be counterintuitive
3. Proper basis: FRE 703
4. Relevant
5. Reliable
6. 403 Balancing
iii. U.S. v. Johnson
1. Expertise not limiting to degrees; a marijuana smokers
testimony allowed in.
iv. Jinro America, Inc. v. Secure Investments, Inc.
1. Expert testimony not allowed in because it was based on
generalizations on Korean business practices.
d. FRE 703 Bases of an Experts Opinion Testimony
i. Expert needs to be made aware of or personally obscure
information; can also observe during trial
ii. Limited to facts/data in forming their opinion (how they normally
conduct their studies)
iii. Point is so expert can apply what they need for their analysis
(reason why hearsay allowed)
iv. Balances based on exclusion (reverse 403 analysis)
e. FRE 704 Opinion on an Ultimate Issue
i. Evidence is not automatically objectionable because it embraces an
ultimate issue
ii. (b) Couldnt say acted w/ or w/o x intent; could say that
presents x characteristic(s)
1. A medical determination is up to the jury (such as the is
insane)
2. ***still a 403 issue evidence has to be helpful; cant say
other witness lying or person raped or victim abused***
iii. Hygh v. Jacobs
1. Professor Cox testimony about the use of force from a fight
crossed the line provided by 704 because he provided a
definition of physical force that essentially concluded the
issue for the jury shouldve been excluded.
f. FRE 705 Disclosing the Facts or Data Underlying an Experts Opinion
i. Unless the court orders otherwise, an expert may state an opinion
and give the reasons for it without first testifying to the
underlying facts or data. But the expert may be required to
disclose those facts or data on cross-examination
g. FRE 706 Court-Appointed Expert Witnesses
i. (a) Appt. Process
ii. (b) Experts Role
iii. (c) Compensation
iv. (d) Disclosing the Appt. to the Jury
v. (e) Parties Choice of Their Own Experts
h. Issues on Reliability
i. How do we know if its junk or junk science vs. reliable evidence
ii. Frye v. U.S. (general acceptance test federally accepted til 92)
1. Systolic blood pressure deception test not yet gained
enough traction to be considered scientific.
iii. Daubert v. Merrell Dow
1. Facts: Dauberts expert has a cutting-edge technique that
disproved Merrells experts; argues that FRE 702 trumps
Frye; Supreme Court agrees and says that general
acceptance not the standard (5 factors to consider)
iv. Daubert factors: applied to 702
1. Whether the technique can be (and has been) tested
2. Whether it has been subjected to peer review and
publication
3. the known or potential rate or error
4. the existence and maintenance or standards controlling the
techniques operation;
5. AND general acceptance in the relevant scientific
community
v. Things to remember:
1. Judge is the gatekeeper
2. Opens the gate to more evidence?
vi. Questions Left Unanswered
1. Standard of Review? (Joiner)
2. Methods and Conclusions? (Joiner)
3. Non-Scientific Experts? (Kuhmo Tire Co. v. Carmichael)
a. Facts: tire failure expert determines that its a
manufacturing defect (used his own 4 factor test).
b. Supreme Court finds that Dauberts general factors
apply to FRE 702 and ultimately excludes the
testimony.
c. Gate-keeping obligation applies to technical and
specialized knowledge
d. Can use Daubert factors however they are not the
holy writ
e. Trial judge has broad latitude
vii. Factors to consider
1. Was information developed for litigation?
2. Was there unjustified extrapolation?
3. Are there other obvious explanations?
4. Is expert being careful?
5. Quantitative restrictions
6. Qualitative restrictions
viii. Test for Reliability of Non-Scientific Evidence
1. Daubert Factors
2. Fyre test
3. Qualifications of the Expert Witness
4. Nothing
ix. Since Kuhmo, harder to get expert evidence into court.
i. The Future
i. Greater role for the Advocates
ii. More motions in limine
iii. More factors to consider
iv. More authority for trial judge
v. Less evidence to the fact finder?
IX. Privileges
a. Federal rules do not really touch on them; punts the issue to the
states/common law
b. Significance of privileges: U.S. asserted state secrets privilege to an
innocent party.
c. Can deny the court of key and significant evidence
i. Pries-penitent; doctor-patient; atty-client
d. You need to know what exact elements are
i. Atty-client
1. Client enjoys; client has to waive
2. Only professional communication
3. Only protects communication, not the facts
e. Attorneys usually hurt themselves when it comes to inadvertent
disclosures.
f. Policy: whatever value would be gained, outweighs the social value
g. Just really have to think about cost-benefit and aware of jurisdictions rules
h. FRE 501 and 502 are the privileges allowed in and the limitations on
inadvertently admitting privileged evidence.
i. General elements:
i. The privilege is the clients
ii. The privilege protects only those confidential communications
made to facilitate professional services
iii. The privilege only protects against confidential communications
iv. The privilege only protects the communications

You might also like